5
Math 206, Spring 2016 Assignment 5 Solutions Due: F ebruary 26, 2016 Par t A. (1) Let w 1  =  1 1 ,  w 2  =  1 2 ,  and  w 3  =  1 0 . Prove that there cannot be  T   L(R 2 , R 3 ) that satises T  ( w 1 ) = 1 2 3 , T  ( w 2 ) = 3 2 5 ,  and  T  ( w 3 ) = 4 1 7 . [Hint : Find two distinct ways to expr ess  0   R 2 as a linear combination of  w 1 , w 2  and  w 3 . If you assume that  T  is linear, what do thes e tw o repre sen tatio ns for  0 tell you about the value of  T (0)?] Solution.  Suppose, for the sake of con tradiction, that such a linear transfo rmation  T  exists. Observe that  0 = 0w 1  + 0 w 2  + 0 w 3 , but also that 0 = 2  1 1  −1 2 3  1 0 = 2w 1  − w 2  − 3w 3 . Now the linearity of  T  tells us that, on the one hand, T (0) =  T (0w 1  + 0w 2  + 0 w 3 ) = 0 1 2 3 + 0 3 2 5 + 0 4 1 7 = 0 0 0 . On the other hand, linearity of  T  gives T (0) =  T (2w 1  − w 2  − 3w 3 ) = 2 1 2 3 1 3 2 5 3 4 1 7 = 13 3 20 . Hence our assumption leads to the conclusion that  T (0) takes two separate values; but this contradicts the fact that  T  is a function. So our initial assumption must be false, and we conclude that there is no T   L(R 2 , R 3 ) that satises the given properties.   (2) Suppose that  T  ∈ L(R c , R r ) is given. Prove that im(T ) satises the following propertie s: (a) The vector  0 ∈  R r is an element of im(T ). Solution.  There ar e a few ways one cou ld approa ch this. Here’s one: since T   L(R c , R r ), there exists some  A  ∈  R r×c so that for all  x ∈  R c we have  T (x) =  Ax. Hence T (0) =  A0 = 0Col 1 (A) + 0Col 2 (A) + ··· + 0Col c (A) =  0. Hence  0 ∈  R r . Here’s a dierent proof. We know that for any  n  ∈  Z + and any  x ∈  R n , we have 0x =  0. Hence by linearity we have T (0) =  T (0x) = 0T (x) =  0 (where the initial  0 we plugged into  T  is an element of  R c , and the latter  0 is an element of  R r ). http://palmer.wellesley.edu/ ~ aschultz/w16/math206  Page 1 of  5

w16_206_hwk05_solns

Embed Size (px)

Citation preview

8/15/2019 w16_206_hwk05_solns

http://slidepdf.com/reader/full/w16206hwk05solns 1/5

Math 206, Spring 2016 Assignment 5 – Solutions Due: February 26, 2016

Part A.

(1) Let

w1  =

  11

,   w2  =

  −1

2

,   and   w3  =

  10

.

Prove that there cannot be  T   ∈ L(R2,R3) that satisfies

T  (w1) =

1

23

, T  (w2) =

3

−25

,   and   T  (w3) =

4

17

.

[Hint: Find two distinct ways to express   0   ∈   R2 as a linear combination of   w1, w2   and   w3. If you

assume that  T   is linear, what do these two representations for   0 tell you about the value of  T (0)?]

Solution.   Suppose, for the sake of contradiction, that such a linear transformation T   exists. Observethat   0 = 0w1 + 0w2 + 0w3, but also that

0 = 2

  11

  −1

2

− 3

  10

= 2w1 − w2 − 3w3.

Now the linearity of  T  tells us that, on the one hand,

T (0) =  T (0w1 + 0w2 + 0w3) = 0

1

23

+ 0

3

−25

+ 0

4

17

=

0

00

.

On the other hand, linearity of  T   gives

T (0) =  T (2w1 − w2 − 3w3) = 2

123

− 1

3−2

5

− 3

417

=

−133

−20

.

Hence our assumption leads to the conclusion that T (0) takes two separate values; but this contradictsthe fact that  T  is a function. So our initial assumption must be false, and we conclude that there is noT   ∈ L(R2,R3) that satisfies the given properties.  

(2) Suppose that T   ∈ L(Rc,Rr) is given. Prove that im(T ) satisfies the following properties:(a) The vector   0 ∈  R

r is an element of im(T ).

Solution.   There are a few ways one could approach this. Here’s one: since T  ∈ L(Rc,Rr), thereexists some A  ∈  R

r×c so that for all   x ∈  Rc we have  T (x) =  Ax. Hence

T (0

) =  A0

 = 0Col1(A) + 0Col2(A) + · · · + 0Colc(A) = 0

.Hence   0 ∈  R

r.

Here’s a different proof. We know that for any  n  ∈  Z+ and any   x ∈  R

n, we have 0x =   0. Henceby linearity we have

T (0) =  T (0x) = 0T (x) =  0

(where the initial   0  we plugged into  T   is an element of  Rc, and the latter   0  is an element of  Rr).

http://palmer.wellesley.edu/~aschultz/w16/math206   Page 1 of  5

8/15/2019 w16_206_hwk05_solns

http://slidepdf.com/reader/full/w16206hwk05solns 2/5

Math 206, Spring 2016 Assignment 5 – Solutions Due: February 26, 2016

(b) If   v, w ∈  im(T ), then   v + w ∈  im(T ).

Solution.   Let   v, w  ∈   im(T ) be given. This means that there exist vectors   x1, x2   ∈  Rc so that

T (x1) =  v  and  T (x2) =  w. But then observe that

T (x1 + x2) =  T (x1) + T (x2) (linearity of   T )

=  v  + w.

Hence   v + w ∈  im(T ), since it is the output of the vector   x1 + x2Rc.  

(c) If   v ∈  im(T ) and  c  ∈  R, then cv ∈  im(T ).

Solution.   Let   v ∈  im(T ) and  c  ∈  R  be given. By the definition of image, this means that thereexists some   x ∈  R

c so that T (x) =  v. Note that we therefore have

T (cx) =  cT (x) (linearity of   T )

= cv

.

Hence cv ∈  im(T ), since it is the output associated to the vector  cx ∈  Rc.  

(3) For each of the following functions, determine (with proof) if the function is linear. If so, give the matrixto which it corresponds.

(a) The function T   :  R3 →  R2 defined by  T 

x

yz

=

  3x + yx + y − 2z

Solution.   We’ll check that T   is linear.

First, let

x =

x1

x2

x3

  and   w =

w1

w2

w3

be given. We then have

T (v + w) =  T 

x1 + w1

x2 + w2

x3 + w3

  (definition of vector addition)

=

  3(x1 + w1) + (x2 + w2)

(x1 + w1) + (x2 + w2) − 2(x3 + w3)

  (definition of  T )

=

  3x1 + x2

x1 + x2 − 2x3

+

  3w1 + w2

w1 + w2 − 2w3

  (definition of vector addition)

= T 

x1

x2

x3

+ T 

w1

w2

w3

  (definition of  T )

= T (x) + T (w).

This verifies the first property of linearity.

http://palmer.wellesley.edu/~aschultz/w16/math206   Page 2 of  5

8/15/2019 w16_206_hwk05_solns

http://slidepdf.com/reader/full/w16206hwk05solns 3/5

Math 206, Spring 2016 Assignment 5 – Solutions Due: February 26, 2016

For the second, let

x =

x1

x2

x3

  and   c ∈  R

be given. Then we have

T (cx) =  T 

cx1

cx2

cx3

  (definition of scaling)

=

  3(cx1) + (cx2)

(cx1) + (cx2) − 2(cx3)

  (definition of  T )

= c

  3x1 + x2

x1 + x2 − 2x3

  (definition of scaling)

= cT (x).

This verifies the second property of linearity.

Now that we know  T  is linear, we can compute the matrix to which it corresponds. By a theoremfrom class, we know this matrix is

  T (e1)   · · ·   T (ec)

=

  3 1 01 1   −2

.

(b) The function T   :  R2 →  R3 defined by  T 

  xy

=

x

1y

Solution.   This is not linear. Note that

  00

=

0

10

.

Of course we know that for   0 ∈  R2 we have   0 + 0 =  0, but observe that

T (0 + 0) =  T (0) =

0

10

whereas

T (0) + T (0) = 01

0

+ 01

0

= 02

0

.

Since T (0 + 0) = T (0) + T (0), we see that T   fails to be linear.  

Part B.

(a) Let   v =

1

−22

,  and for a given   w ∈  R

3 define   w⊥ =  w − proj

vw.

http://palmer.wellesley.edu/~aschultz/w16/math206   Page 3 of  5

8/15/2019 w16_206_hwk05_solns

http://slidepdf.com/reader/full/w16206hwk05solns 4/5

Math 206, Spring 2016 Assignment 5 – Solutions Due: February 26, 2016

(i) Prove that  w⊥ is orthogonal to   v.

Solution.   We will show that   w⊥ · v = 0. To help in this calculation, recall that proj

vw =

w·v

v·vv

. Hence we getw⊥ · v =

w −

  w · v

v · v v

· v

=  w · v −  w · v

v · v v · v   (linearity of dot product)

=  w · v − w · v   (cancellation of   v · v)

= 0.

(ii) Prove that the function  T   :  R3 →  R3 given by T (w) =  w

⊥ is a linear transformation.

Solution.

  We will check the relevant axioms. First, suppose that  w

1  and  w

2  are given inR3. Then we have

T (w1 + w2) = (w1 + w2) − projv

(w1 + w2) (definition of   T )

= (w1 + w2) − projv

w1 − projv

w2   (linearity of proj from class)

= (w1 − projv

w1) + (w2 − projv

w2   commutativity of vector addition)

= T (w1) + T (w2).

This verifies the first condition of linearity.

For the second, let   w ∈  R3 and c  ∈  R  be given. Then we have

T (cw) = (cw) − projv

(cw)(definition of  T )

= cw − cprojv

w   (linearity of proj from class)

= c(w − projv

w) (scaling distributes across addition)

= cT (w) (definition of   T ).

This verifies the second condition of linearity.  

(iii) Compute the matrix that corresponds to  T .

Solution.   By a theorem from class, we know this matrix is   T (e1)   · · ·   T (ec)

=

  e1 −   e1·v

v·vv e2 −   e2·v

v·vv e3 −   e3·v

v·vv

=

8/9 2/9   −2/9

2/9 5/9 4/9−2/9 4/9 5/9

.

(iv) Is T   injective?

Solution.   We’ll compute the reduced row echelon form of the matrix associated to  T : 1 0   −1/2

0 1 10 0 0

.

(You can reach this form by performing the following sequence of operations:   9

8ρ1, −2

9ρ1 +

ρ2,   29

ρ1  +  ρ3, 2ρ2, −1

2ρ2 +  ρ3, −1

4ρ2 +  ρ1.) Note that this matrix has rank 2. By a theorem

http://palmer.wellesley.edu/~aschultz/w16/math206   Page 4 of  5

8/15/2019 w16_206_hwk05_solns

http://slidepdf.com/reader/full/w16206hwk05solns 5/5

Math 206, Spring 2016 Assignment 5 – Solutions Due: February 26, 2016

from class, since the rank is less than the number of columns, we have that  T  is not injective.

(b) Suppose that   v ∈  span {w1, w2, · · ·  , wk}. Prove that

span {w1, w2, · · ·  , wk} = span {w1, w2, · · ·  , wk, v} .

Solution.   We’ll prove this result using a dual containment argument. First, we argue that{w1, w2, · · ·  , wk} ⊆  span {w1, w2, · · ·  , wk, v}. So let   z  ∈ {w1, w2, · · ·  , wk}  be given; our goal isto show that   z ∈  span {w1, w2, · · ·  , wk, v}. By definition of span, our hypothesis tells us that   z isa linear combination of the vectors  w1, · · ·  , wk. This means that there exist  d1, · · ·  , dk  ∈  R  so that

z =  d1w1 + · · · + dkwk.

But observe that then we also have

z =  d1w1 + · · · + dkwk + 0 =  d1w1 + · · · + dkwk + 0v.

Hence we have exhibited   z  as a linear combination of the vectors   w1, · · ·  , wk, v. Therefore   z  ∈span {w1, · · ·  , wk, v}. This verifies the first containment.

For the containment span {w1, w2, · · ·  , wk, v} ⊆ {w1, w2, · · ·  , wk}, suppose that z ∈  span {w1, · · ·  , wk, v

This means that  z  is a linear combination of  w1, · · ·  , wk, v, and hence (by definition of linear com-bination) there exist d1, · · ·  , dk, dk+1  ∈  R  so that

z =  d1w1 + · · · + dkwk + dk+1v.

Furthermore, we’re told that v ∈  span {w1, · · ·  , wk}. Hence v is a linear combination of w1, · · ·  , wk,and so there are real numbers  c1, · · ·  , ck  satisfying

v =  c1w1 + · · · + ckwk.

We plug this expression for   v   into our expression for   z  above to find

z =  d1w1 + · · · + dkwk + dk+1v

= d1w

1 + · · · + dkw

k + dk+1(c1w

1 + · · · + ckw

k)= (d1 + dk+1c1)w1 + · · · + (dk + dk+1ck)wk.

Now since each of the   di   and   ci   are real numbers, each coefficient   di  + dk+1ci   above is a realnumber. So we have exhibited   z  as a linear combination of   w1, · · ·  , wk. Therefore we have   z  ∈span {w1, · · ·  , wk}. This verifies the second containment.  

http://palmer.wellesley.edu/~aschultz/w16/math206   Page 5 of  5